Given three positive numbers $a,b,c$. Prove that $\sum\limits_{cyc}\sqrt{\frac{a+b}{b+1}}\geqq3\sqrt[3]{\frac{4\,abc}{3\,abc+1}}$ .

holder-inequalityinequalitymuirhead-inequalitysubstitutionuvw

Ji Chen. Given three positive numbers $a, b, c$. Prove that
$$\sum\limits_{cyc}\sqrt{\frac{a+ b}{b+ 1}}\geqq 3\sqrt[3]{\frac{4\,abc}{3\,abc+ 1}}$$

Of course, we've to solve it by $uvw$, before that, I tried to use Holder-inequality with integer polynomials but without a high probability of success for me against this particular problem …

I found here: https://artofproblemsolving.com/community/c6h538065p3209975, something obvious

Best Answer

By AM-GM $$\sum_{cyc}\sqrt{\frac{a+b}{b+1}}\geq3\sqrt[6]{\prod\limits_{cyc}\frac{a+b}{a+1}}.$$ Thus, it's enough to prove that $$(a+b)(a+c)(b+c)(3abc+1)^2\geq16a^2b^2c^2(a+1)(b+1)(c+1).$$ Now, let $a+b+c=3u$, $ab+ac+bc=3v^2$ and $abc=w^3$.

Thus, we need to prove that $$(9uv^2-w^3)(3w^3+1)^2\geq16w^6(w^3+3v^2+3u+1)$$ and since by AM-GM $uv^2\geq w^3,$ it's enough to prove that $$uv^2(3w^3+1)^2\geq2w^6(w^3+3v^2+3u+1),$$ which is true by AM-GM.

Indeed, by AM-GM $$2w^6(w^3+3v^2+3u+1)=2w^9+6v^2w^6+6uw^6+2w^6\leq$$ $$\leq2uv^2w^6+6uv^2w^5+6uv^2w^4+2uv^2w^3$$ and it's enough to prove that: $$(3w^3+1)^2\geq2w^6+6w^5+6w^4+2w^3$$ or $$(3w^3+1)^2\geq2w^3(w+1)^3.$$ Can you end it now?